Answers and Hints




  1. Prime Numbers
    2.
    They are $ 2, 3, 5, 7, 11, 13, 17, 19, 23, 29, 31, 37, 41, 43, 47, 53, 59,$
    $ 61, 67, 71, 73, 79, 83, 89, 97$ .
    3.
    Emulate the proof of Proposition 1.2.6.




  2. The Ring of Integers Modulo $ n$
    2.
    They are $ 5$ , $ 13$ , $ 3$ , and $ 8$ .
    3.
    For example $ x=22$ , $ y=-39$ .

    4.
    Hint: Use the binomial theorem and prove that if $ r\geq 1$ then $ p$ divides $ \binom{p}{r}$ .

    7.
    For example, $ S_1 = \{0,1,2,3,4,5,6\}$ , $ S_2 = \{1,3,5,7,11,13,23\}$ , $ S_3 = \{0,2,4,6,8,10,12\}$ , and $ S_4 = \{2,3,5,7,11,13,29\}.$ In each we find $ S_i$ by listing the first seven numbers satisfying the $ i$ th condition, then adjusted the last number if necessary so that the reductions would be distinct modulo $ 7$ .
    8.
    An integer is divisible by $ 5$ if and only if the last digits is 0 or $ 5$ . An integer is divisible by $ 9$ if and only if the sum of the digits is divisible by $ 9$ . An integer is divisible by $ 11$ if and only if the alternating sum of the digits is divisible by $ 11$ .
    9.
    Hint for part (a): Use the divisibility rule you found in Exercise 1.8.

    10.
    $ 71$

    11.
    $ 8$

    12.
    As explained on page [*], we know that $ \mathbb{Z}/n\mathbb{Z}{}$ is a ring for any $ n$ . Thus to show that $ \mathbb{Z}/p\mathbb{Z}{}$ is a field it suffices to show that every nonzero element $ \overline{a}\in\mathbb{Z}/p\mathbb{Z}{}$ has an inverse. Lift $ a$ to an element $ a\in\mathbb{Z}$ , and set $ b=p$ in Proposition 2.3.1. Because $ p$ is prime, $ \gcd(a,p)=1$ , so there exists $ x,y$ such that $ ax + py = 1$ . Reducing this equality modulo $ p$ proves that $ \overline{a}$ has an inverse $ x\pmod{p}$ . Alternative one could argue just like after Definition 2.1.15 that $ \overline{a}^m=1$ for some $ m$ , so some power of $ \overline{a}$ is the inverse of $ \overline{a}$ .

    13.
    $ 302$

    15.
    Only for $ n=1,2$ . If $ n>2$ , then $ n$ is either divisible by an odd prime $ p$ or $ 4$ . If $ 4\mid n$ , then $ 2^e-2^{e-1}$ divides $ \varphi (n)$ for some $ e\geq 2$ , so $ \varphi (n)$ is even. If an odd $ p$ divides $ n$ , then the even number $ p^e-p^{e-1}$ divides $ \varphi (n)$ for some $ e\geq 1$ .

    16.
    The map $ \psi$ is a homomorphism since both reduction maps

    $\displaystyle \mathbb{Z}/mn\mathbb{Z}{}\to \mathbb{Z}/m\mathbb{Z}{}$   and$\displaystyle \quad
\mathbb{Z}/mn\mathbb{Z}{}\to \mathbb{Z}/n\mathbb{Z}{}$

    are homomorphisms. It is injective because if $ a\in\mathbb{Z}$ is such that $ \psi(a)=0$ , then $ m\mid a$ and $ n\mid a$ , so $ mn\mid a$ (since $ m$ and $ n$ are coprime), so $ a\equiv 0\pmod{mn}$ . The cardinality of $ \mathbb{Z}/mn\mathbb{Z}{}$ is $ mn$ and the cardinality of the product $ \mathbb{Z}/m\mathbb{Z}{}\times \mathbb{Z}/n\mathbb{Z}{}$ is also $ mn$ , so $ \psi$ must be an isomorphism. The units $ (\mathbb{Z}/mn\mathbb{Z}{})^*$ are thus in bijection with the units $ (\mathbb{Z}/m\mathbb{Z}{})^* \times (\mathbb{Z}/n\mathbb{Z}{})^*$ .

    For the second part of the exercise, let $ g=\gcd(m,n)$ and set $ a=mn/g$ . Then $ a\not\equiv 0\pmod{mn}$ , but $ m\mid a$ and $ n\mid a$ , so $ a\ker(\psi)$ .

    17.
    We express the question as a system of linear equations modulo various numbers, and use the Chinese remainder theorem. Let $ x$ be the number of books. The problem asserts that

    $\displaystyle x$ $\displaystyle \equiv 6\pmod{7}$    
    $\displaystyle x$ $\displaystyle \equiv 2\pmod{6}$    
    $\displaystyle x$ $\displaystyle \equiv 1\pmod{5}$    
    $\displaystyle x$ $\displaystyle \equiv 0\pmod{4}$    

    Applying CRT to the first pair of equations we find that $ x\equiv 20\pmod{42}$ . Applying CRT to this equation and the third we find that $ x\equiv 146\pmod{210}$ . Since $ 146$ is not divisible by $ 4$ , we add multiples of $ 210$ to $ 146$ until we find the first $ x$ that is divisible by $ 4$ . The first multiple works, and we find that the aspiring mathematicians have $ 356$ math books.

    18.
    Note that $ p=3$ works, since $ 11=3^2+2$ is prime. Now suppose $ p\neq $ is any prime such that $ p$ and $ p^2+2$ are both prime. We must have $ p\equiv 1\pmod{3}$ or $ p\equiv 2\pmod{3}$ . Then $ p^2\equiv 1\pmod{3}$ , so $ p^2+2\equiv 0\pmod{3}$ , which contradicts the fact that $ p^2+2$ is prime.

    19.
    For (a) $ n=1,2$ , see solution to Exercise 2.15. For (b), yes there are many such examples. For example, $ m=2$ , $ n=4$ .

    20.
    By repeated application of multiplicativity and Equation (2.2.2) on page [*], we see that if $ n=\prod_i p_i^{e_i}$ is the prime factorization of $ n$ , then

    $\displaystyle \varphi (n) = \prod_i (p_i^{e_i} - p_i^{e_i-1})
= \prod_i p_i^{e_i-1} \cdot \prod_i (p_i-1).
$

    23.
    1, 6, 29, 34

    24.
    Let $ g=\gcd(12n+1,30n+2)$ . Then $ g\mid 30n+2 - 2\cdot (12n+1) = 6n$ . For the same reason $ g$ also divides $ 12n+1 - 2\cdot (6n) = 1$ , so $ g=1$ , as claimed.

    27.
    There is no primitive root modulo $ 8$ , since $ (\mathbb{Z}/8\mathbb{Z}{})^*$ has order $ 4$ , but every element of $ (\mathbb{Z}/8\mathbb{Z}{})^*$ has order $ 2$ . Prove that if $ \zeta$ is a primitive root modulo $ 2^n$ , for $ n\geq 3$ , then the reduction of $ \zeta$ mod $ 8$ is a primitive root, a contradiction.

    28.
    $ 2$ is a primitive root modulo $ 125$ .

    29.
    Let $ \prod_{i=1}^m p_i^{e_i}$ be the prime factorization of $ n$ . Slightly generalizing Exercise 16 we see that

    $\displaystyle (\mathbb{Z}/n\mathbb{Z}{})^* \cong \prod (\mathbb{Z}/p_i^{e_i}\mathbb{Z}{})^*.
$

    Thus $ (\mathbb{Z}/n\mathbb{Z}{})^*$ is cyclic if and only if the product $ (\mathbb{Z}/p_i^{e_i}\mathbb{Z}{})^*$ is cyclic. If $ 8\mid n$ , then there is no chance $ (\mathbb{Z}/n\mathbb{Z}{})^*$ is cyclic, so assume $ 8\nmid n$ . Then by Exercise 2.28 each group $ (\mathbb{Z}/p_i^{e_i}\mathbb{Z}{})^*$ is itself cyclic. A product of cyclic groups is cyclic if and only the orders of the factors in the product are coprime (this follows from Exercise 2.16). Thus $ (\mathbb{Z}/n\mathbb{Z}{})^*$ is cyclic if and only if the numbers $ p_i(p_i-1)$ , for $ i=1,\ldots, m$ are pairwise coprime. Since $ p_i-1$ is even, there can be at most one odd prime in the factorization of $ n$ , and we see that $ (\mathbb{Z}/n\mathbb{Z}{})^*$ is cyclic if and only if $ n$ is an odd prime power, twice an odd prime power, or $ n=4$ .




  3. Public-Key Cryptography
    1.
    The best case is that each letter is A. Then the question is to find the largest $ n$ such that $ 1 + 27 + \cdots + 27^n \leq 10^{20}$ . By computing $ \log_{27}(10^{20})$ , we see that $ 27^{13}<10^{20}$ and $ 27^{14}>10^{20}$ . Thus $ n\leq 13$ , and since $ 1 + 27 + \cdots + 27^{n-1} < 27^n$ , and $ 2\cdot 27^{13}<10^{20}$ , it follows that $ n=13$ .

    2.
    This is not secure, since it is just equivalent to a ``Ceaser Cipher'', that is a permutation of the letters of the alphabet, which is well-known to be easily broken using a frequency analysis.

    3.
    If we can compute the polynomial

    $\displaystyle f = (x-p)(x-q)(x-r)
= x^3 - (p+q+r)x^2 + (pq+pr+qr)x - pqr,
$

    then we can factor $ n$ by finding the roots of $ f$ , e.g., using Newton's method (or Cardona's formula for the roots of a cubic). Because $ p$ , $ q$$ r$ , are distinct odd primes we have

    $\displaystyle \varphi (n) =(p-1)(q-1)(r-1) = pqr-(pq+pr+qr)+p+q+r,$

    and

    $\displaystyle \sigma(n) = 1 + (p+q+r)+(pq+pr+qr) + pqr.$

    Since we know $ n$ , $ \varphi (n)$ , and $ \sigma(n)$ , we know

    $\displaystyle \sigma(n)-1-n$ $\displaystyle = (p+q+r)+(pq+pr+qr),$   and    
    $\displaystyle \varphi (n)-n$ $\displaystyle = (p+q+r)-(pq+pr+qr).$    

    We can thus compute both $ p+q+r$ and $ pq+pr+qr$ , hence deduce $ f$ and find $ p,q,r$ .




  4. Quadratic Reciprocity
    1.
    They are all $ 1$ , $ -1$ , 0 , and $ 1$ .

    3.
    By Proposition 4.3.4 the value of $ \left(\frac{3}{p}\right)$ depends only on the reduction $ \pm p\pmod{12}$ . List enough primes $ p$ such that the $ \pm p$ reduce to $ 1,5,7,11$ modulo $ 12$ and verify that the asserted formula holds for each of them.

    7.
    Since $ p=2^{13}-1$ is prime there are either two solutions or no solutions to $ x^2\equiv 5\pmod{p}$ , and we can decide which using quadratic reciprocity. We have

    $\displaystyle \left(\frac{5}{p}\right) = (-1)^{(p-1)/2\cdot (5-1)/2}\left(\frac{p}{5}\right) = \left(\frac{p}{5}\right),$

    so there are two solutions if and only if $ p=2^{13}-1$ is $ \pm 1$ mod $ 5$ . In fact $ p\equiv 1\pmod{5}$ , so there are two solutions.

    8.
    We have $ 4^{48} = 2^{96}$ . By Fermat's Little Theorem $ 2^{96}=1$ , so $ x=1$ .

    9.
    For (a) take $ a=19$ and $ n=20$ . We found this example using the Chinese remainder theorem applied to $ 4\pmod{5}$ and $ 3\pmod{4}$ , and used that $ \left(\frac{19}{20}\right) = \left(\frac{19}{5}\right)\cdot\left(\frac{19}{4}\right) = (-1)(-1)=1$ , yet $ 19$ is not a square modulo either $ 5$ or $ 4$ , so is certainly not a square modulo $ 20$ .

    10.
    Hint: First reduce to the case that $ 6k-1$ is prime, by using that if $ p$ and $ q$ are primes not of the form $ 6k-1$ , then neither is their product. If $ p=6k-1$ divides $ n^2+n+1$ , it divides $ 4n^2+4n+4 = (2n+1)^2+3$ , so $ -3$ is a quadratic residue modulo $ p$ . Now use quadratic reciprocity to show that $ -3$ is not a quadratic residue modulo $ p$ .




  5. Continued Fractions
    9.
    Suppose $ n=x^2 + y^2$ , with $ x,y\in\mathbb{Q}$ . Let $ d$ be such that $ dx, dy\in\mathbb{Z}$ . Then $ d^2 n = (dx)^2 + (dy)^2$ is a sum of two integer squares, so by Theorem 5.6.1 if $ p\mid d^2
n$ and $ p\equiv 3 \pmod{4}$ , then $ \ord _p(d^2n)$ is even. We have $ \ord _p(d^2n)$ is even if and only if $ \ord _p(n)$ is even, so Theorem 5.6.1 implies that $ n$ is also a sum of two squares.

    11.
    The squares modulo $ 8$ are $ 0,1,4$ , so a sum of two squares reduces modulo $ 8$ to one of $ 0,1,2,4$ or $ 5$ . Four consecutive integers that are sums of squares would reduce to four consecutive integers in the set $ \{0,1,2,4,5\}$ , which is impossible.




  6. Elliptic Curves
    2.
    The second point of intersection is $ (129/100, 383/1000)$ .

    3.
    The group is cyclic of order $ 9$ , generated by $ (4,2)$ . The elements of $ E(K)$ are

    $\displaystyle \{\O, (4,2), (3,4), (2,4), (0,4), (0,1), (2,1), (3,1), (4,3)\}.$

    4.
    In part (a) the pattern is that $ N_p=p+1$ . For part (b), a hint is that when $ p\equiv 2\pmod{3}$ , the map $ x\mapsto x^3$ on $ (\mathbb{Z}/p\mathbb{Z}{})^*$ is an automorphism, so $ x\mapsto x^3+1$ is a bijection. Now use what you learned about squares in $ \mathbb{Z}/p\mathbb{Z}{}$ from Chapter 4.

    5.
    For all sufficiently large real $ x$ , the equation $ y^2=x^3+ax+b$ has a real solution $ y$ . Thus the group $ E(\mathbb{R})$ is not countable, since $ \mathbb {R}$ is not countable. But any finitely generated group is countable.

    6.
    In a course on abstract algebra one often proves the nontrivial fact that every subgroup of a finitely generated abelian group is finitely generated. In particular, the torsion subgroup $ G_{\tor }$ is finitely generated. However, a finitely generated abelian torsion group is finite.

    7.
    Hint: Multiply both sides of $ y^2=x^3+ax+b$ by a power of a common denominator, and ``absorb'' powers into $ x$ and $ y$ .

    8.
    Hint: see Exercise 4.6.

William 2007-06-01